0 Daumen
453 Aufrufe


ich habe folgendes Problem. Ich kann die Abschätzung der folgenden Gleichung nicht nachvollziehen bzw. würde gerne wissen wie man das berechnet oder darauf kommt:

$$P\{\max\limits_{i\leq n}|S_i|\ge 3\lambda\sqrt{n}\} \leq P\{|S_n|\ge \lambda\sqrt{n}\}+\sum\limits_{i=1}^{n-1}P(E_i\cap\{|S_n-S_i|\ge 2\lambda\sqrt{n}\})$$

Weiter ist gegeben, dass

$$E_i=\{\max\limits_{j<i}|S_j|<3\lambda\sqrt{n}\leq |S_i|\}$$

gilt.

Es gilt ja

$$P\{\max\limits_{i\leq n}|S_i|\ge 3\lambda\sqrt{n}\}=P(\cup E_i)$$

Wie kommt man dann weiter?

Avatar von

Ein anderes Problem?

Stell deine Frage

Willkommen bei der Mathelounge! Stell deine Frage einfach und kostenlos

x
Made by a lovely community